Last visit was: 02 May 2024, 22:33 It is currently 02 May 2024, 22:33

Close
GMAT Club Daily Prep
Thank you for using the timer - this advanced tool can estimate your performance and suggest more practice questions. We have subscribed you to Daily Prep Questions via email.

Customized
for You

we will pick new questions that match your level based on your Timer History

Track
Your Progress

every week, we’ll send you an estimated GMAT score based on your performance

Practice
Pays

we will pick new questions that match your level based on your Timer History
Not interested in getting valuable practice questions and articles delivered to your email? No problem, unsubscribe here.
Close
Request Expert Reply
Confirm Cancel
SORT BY:
Date
Math Revolution GMAT Instructor
Joined: 16 Aug 2015
Posts: 10160
Own Kudos [?]: 16634 [1]
Given Kudos: 4
GMAT 1: 760 Q51 V42
GPA: 3.82
Send PM
Math Revolution GMAT Instructor
Joined: 16 Aug 2015
Posts: 10160
Own Kudos [?]: 16634 [0]
Given Kudos: 4
GMAT 1: 760 Q51 V42
GPA: 3.82
Send PM
Math Revolution GMAT Instructor
Joined: 16 Aug 2015
Posts: 10160
Own Kudos [?]: 16634 [0]
Given Kudos: 4
GMAT 1: 760 Q51 V42
GPA: 3.82
Send PM
Math Revolution GMAT Instructor
Joined: 16 Aug 2015
Posts: 10160
Own Kudos [?]: 16634 [0]
Given Kudos: 4
GMAT 1: 760 Q51 V42
GPA: 3.82
Send PM
Re: The Ultimate Q51 Guide [Expert Level] [#permalink]
Expert Reply
The probability that event A occurs is 0.5 and the probability that event B occurs is 0.7. What is the probability that event A occurs but not event B?

1) Event A and event B are independent.
2) The probability that neither event A nor event B occurs is 0.15.

=>Condition 1)
We call an event A and an event B independent if P(A∩B) = P(A)*P(B).
P(A∩B) = P(A)*P(B) = 0.5 * 0.7 = 0.35.
P(A∩B^C) = P(A) - P(A∩B) = 0.5 – 0.35 = 0.15.
This is sufficient.

Condition 2)
1 – P(A∪B) = 0.15. It means P(A∪B) = 0.85.
P(A∪B) = P(A) + P(B) - P(A∩B) = 0.5 + 0.7 - P(A∩B) = 0.85.
Thus P(A∩B) = 0.15.
P(A∩B^C) = P(A) - P(A∩B) = 0.5 – 0.35 = 0.15.
This is also sufficient too.

Ans: D
Math Revolution GMAT Instructor
Joined: 16 Aug 2015
Posts: 10160
Own Kudos [?]: 16634 [0]
Given Kudos: 4
GMAT 1: 760 Q51 V42
GPA: 3.82
Send PM
Re: The Ultimate Q51 Guide [Expert Level] [#permalink]
Expert Reply
If 3^x>10, which of the following must be true?

I. x>2 II. x>3 III. x>4

A. I only
B. II only
C. III only
D. I and II only
E. I, II, and III


=> 3^x > 10 > 3^2
Thus x > 2

Ans: A
Math Revolution GMAT Instructor
Joined: 16 Aug 2015
Posts: 10160
Own Kudos [?]: 16634 [0]
Given Kudos: 4
GMAT 1: 760 Q51 V42
GPA: 3.82
Send PM
Re: The Ultimate Q51 Guide [Expert Level] [#permalink]
Expert Reply
Which of the following inequalities is equal to |x-3|x||<8?

A. 0<x<4
B. 0<x<2
C. -2<x<4
D. -2<x<2
E. -2<x<0

=> -8 < x-3|x| <8

1) x >= 0
-8 < x-3x < 8
-8 < -2x < 8
-4 < x < 4
0 <= x < 4


2) x < 0
-8 < x-3|x| < 8
-8 < x+3x < 8
-8 < 4x < 8
-2 < x < 2
-2 < x < 0

Thus, -2 < x < 4

Ans: C
Math Revolution GMAT Instructor
Joined: 16 Aug 2015
Posts: 10160
Own Kudos [?]: 16634 [0]
Given Kudos: 4
GMAT 1: 760 Q51 V42
GPA: 3.82
Send PM
Re: The Ultimate Q51 Guide [Expert Level] [#permalink]
Expert Reply
[GMAT math practice questions]

(1/11)+(1/12)+……+(1/19)+(1/20) is including in which of the following ranges?

A. 0 ~ (1/5)
B. (1/5) ~ (1/4)
C. (1/4)~(1/3)
D. (1/3)~(1/2)
E. (1/2)~1

=> (1/20) < (1/11), (1/20) < (1/12), …, (1/20) < (1/19).
(1/20) + (1/20) + … + (1/20) < (1/11)+(1/12)+……+(1/19)+(1/20)
(1/20)*10 < (1/11)+(1/12)+……+(1/19)+(1/20)
(1/2) < (1/11)+(1/12)+……+(1/19)+(1/20)

(1/11) < (1/10), (1/12) < (1/10), … , (1/20) < (1/10)
(1/11)+(1/12)+……+(1/19)+(1/20) < (1/10) + (1/10) + … + (1/10) = 1

Thus, 1/2 < (1/11)+(1/12)+……+(1/19)+(1/20) < 1

Ans: E
Math Revolution GMAT Instructor
Joined: 16 Aug 2015
Posts: 10160
Own Kudos [?]: 16634 [0]
Given Kudos: 4
GMAT 1: 760 Q51 V42
GPA: 3.82
Send PM
Re: The Ultimate Q51 Guide [Expert Level] [#permalink]
Expert Reply
If n is the sum of the first 40 positive integers, what is the greatest prime factor of n?

A. 29
B. 31
C. 37
D. 41
E. 43

1 + 2 + … + 40 = (40*41)/2 = 2^3*5*41
Thus the greatest prime factor of n is 41.

Ans: D
Math Revolution GMAT Instructor
Joined: 16 Aug 2015
Posts: 10160
Own Kudos [?]: 16634 [0]
Given Kudos: 4
GMAT 1: 760 Q51 V42
GPA: 3.82
Send PM
Re: The Ultimate Q51 Guide [Expert Level] [#permalink]
Expert Reply
[GMAT math practice question]

Which of the following is the closest to 11*10^{10}–4*10^4?

A. 10^2
B. 10^7
C. 10^8
D. 10^9
E. 10^11

=>
4*10^4 is relative small compared with 11*10^{10}. Thus we can consider 11*10^{10} only. It is close 10^{11}.

Ans: E
Math Revolution GMAT Instructor
Joined: 16 Aug 2015
Posts: 10160
Own Kudos [?]: 16634 [0]
Given Kudos: 4
GMAT 1: 760 Q51 V42
GPA: 3.82
Send PM
Re: The Ultimate Q51 Guide [Expert Level] [#permalink]
Expert Reply
If 3^x-3^{x-2}=648, x=?

A. 5
B. 6
C. 7
D. 8
E. 9

=>
3^x-3^{x-2} = 3^{x-2}3^2-3^{x-2} = 3^{x-2}(3^2-1) = 3^{x-2} x 8 = 648
3^{x-2} = 81 = 3^4
x – 2 = 4
x = 6

Ans: B
Math Revolution GMAT Instructor
Joined: 16 Aug 2015
Posts: 10160
Own Kudos [?]: 16634 [0]
Given Kudos: 4
GMAT 1: 760 Q51 V42
GPA: 3.82
Send PM
Re: The Ultimate Q51 Guide [Expert Level] [#permalink]
Expert Reply
[GMAT math practice question]

If the gross profit of a certain company is 20 percent of the revenue from sales of the company, what percent of the cost is the gross profit?

A. 20%
B. 25%
C. 30%
D. 35%
E. 40%

=>
Let R be the revenue of the company. The profit is 0.2*R and the cost is 0.8*R.
The answer is (0.2*R) / (0.8*R) * 100 = 0.25 * 100 = 25(%).

Ans: B
avatar
Intern
Intern
Joined: 11 Jun 2017
Posts: 2
Own Kudos [?]: 0 [0]
Given Kudos: 4
Send PM
Re: The Ultimate Q51 Guide [Expert Level] [#permalink]
MathRevolution wrote:
Even for the mistake type 4 questions, they are becoming more complicated unlike simple questions in the past. For a question like the below, 1)=2), which is no and sufficient. Therefore, D is the answer.

(ex 3) If n is positive integer, is 3n+n2+1 not divisible by 3?
1) n is not multiple of 2
2) n is not multiple of 3

Answer: D


That is, questions are evolving like highly challenging questions, in which the mistake type 4 and 1 are combined. So, we should keep up with the evolving questions. In particular, integer questions are numerously given and you should carefully approach the questions, which will lead you to hit 51.



I agree with your answer but I think the answer can be obtained without any condition (ie) 3n + n2 + 1 is always not divisible by 3 and will leave remainder of 1 or 2. Is my solution right? And will GMAT give questions wherein the answer can be directly deduced?
Math Revolution GMAT Instructor
Joined: 16 Aug 2015
Posts: 10160
Own Kudos [?]: 16634 [0]
Given Kudos: 4
GMAT 1: 760 Q51 V42
GPA: 3.82
Send PM
Re: The Ultimate Q51 Guide [Expert Level] [#permalink]
Expert Reply
Is a positive integer x an odd number?

1) The smallest prime factor of x is 7.
2) The greatest prime factor of x is 7.

=>
1) Since the smallest prime factor of x is 7, it can’t have a prime factor 2.
Thus it should be odd.
This is sufficient.

2) Since the greatest prime factor of x is 7, it may have 2 as a prime factor or it may not.
Thus it may be odd or may be odd.
This is not sufficient.

Ans: A
Intern
Intern
Joined: 07 Sep 2017
Posts: 23
Own Kudos [?]: 0 [0]
Given Kudos: 1
Send PM
Re: The Ultimate Q51 Guide [Expert Level] [#permalink]
MathRevolution wrote:
If a, b, and c are positive integers, is (a+b)c divisible by 3?

1) 2-digit integer ab is divisible by 3.
2) When c is divided by 3, the remainder is 0.

==> If you modify the original condition and the question, in order to have (a+b)c to be divided by 3, a+b or c has to be divided by 3. However, if you look at con 2), c is divisible by 3, hence it is yes and sufficient. For con 1), ab is also divisible by 3, and thus a+b is also divisible by 3, hence it is yes and sufficient. Therefore, the answer is D. This type of question is a 5051-level question which applies CMT 4 (B: if you get A or B too easily, consider D).

Answer: D
Hi, I didn't get you when you said, "This type of question is a 5051-level question which applies CMT 4 (B: if you get A or B too easily, consider D)."
Thanks.


Sent from my iPhone using GMAT Club Forum mobile app
Math Revolution GMAT Instructor
Joined: 16 Aug 2015
Posts: 10160
Own Kudos [?]: 16634 [0]
Given Kudos: 4
GMAT 1: 760 Q51 V42
GPA: 3.82
Send PM
Re: The Ultimate Q51 Guide [Expert Level] [#permalink]
Expert Reply
Is a positive integer x divisible by 12 ?

1) x is divisible by 8
2) x is divisible by 6

=>
1) Since 8 is not a multiple of 12, this is not sufficient.

2) Since 6 is not a multiple of 12, this is not sufficient.

1) & 2) Since lcm(8,6) = 24 is a multiple of 12, these are sufficient.

Ans: C
Math Revolution GMAT Instructor
Joined: 16 Aug 2015
Posts: 10160
Own Kudos [?]: 16634 [0]
Given Kudos: 4
GMAT 1: 760 Q51 V42
GPA: 3.82
Send PM
Re: The Ultimate Q51 Guide [Expert Level] [#permalink]
Expert Reply
[GMAT math practice question]

I. 9/10, 10/11, 11/12, 12/13, 13/14
II. 10/9, 11/10, 12/11, 13/12, 14/13
III. 6/10, 7/10, 8/10, 9/10, 10/10

For which of the following options is the average (arithmetic mean) of the numbers less than the median of numbers?

A. I. only
B. II. only
C. III. only
D. II and III
E. I, II and III

=>

I. Differences between consecutive terms are 10/11 – 9/10 = 1/110, 11/12 – 10/11 = 1/132, 12/13 – 11/12 = 1/156, 13/14 – 12/13 = 1/182. The differences are getting smaller. The average is smaller than the median.

II. Difference between consecutive terms are 11/10 – 10/9 = -1/90, 12/11 – 11/10 = -1/110, 13/12 – 12/11 = -1/132, 14/13 – 13/12 = -1/156. The differences are getting bigger. The average is bigger than the media.

II Difference between consecutive terms are 7/10 – 6/10 = 1/10, 8/10 – 7/10 = 1/10, 9/10 – 8/10 = 1/10, 10/10 – 9/10 = 1/10. All differences are equal. The average is equal to the median.

Ans: A
Math Revolution GMAT Instructor
Joined: 16 Aug 2015
Posts: 10160
Own Kudos [?]: 16634 [0]
Given Kudos: 4
GMAT 1: 760 Q51 V42
GPA: 3.82
Send PM
Re: The Ultimate Q51 Guide [Expert Level] [#permalink]
Expert Reply
[GMAT math practice question]

3 girls and 4 boys line up in a row. If girls stand at the first and boys stand at the last, how many possible cases are there?

A. 96
B. 108
C. 144
D. 192
E. 256

=>

The number of ways that 3 girls permute is 3! = 6 and the number of ways that 4 boys permute is 4! = 24. Those cases happen together and so the number of cases that 3 girls stand at the first and 4 boys stand at the last is 3! * 4! = 6 * 24 = 144.

Ans: C
Math Revolution GMAT Instructor
Joined: 16 Aug 2015
Posts: 10160
Own Kudos [?]: 16634 [0]
Given Kudos: 4
GMAT 1: 760 Q51 V42
GPA: 3.82
Send PM
Re: The Ultimate Q51 Guide [Expert Level] [#permalink]
Expert Reply
[GMAT math practice question]

If x^3y^4z^5<0, is xyz>0?

1) y<0
2) x<0

=>
x^3y^4z^5 < 0 is equivalent to xz < 0. Then the question xyz > 0 is equivalent to y < 0.
The condition 1) is sufficient. And the condition 2) is not sufficient.

Ans: A
Math Revolution GMAT Instructor
Joined: 16 Aug 2015
Posts: 10160
Own Kudos [?]: 16634 [0]
Given Kudos: 4
GMAT 1: 760 Q51 V42
GPA: 3.82
Send PM
Re: The Ultimate Q51 Guide [Expert Level] [#permalink]
Expert Reply
[GMAT math practice question]

Is x/y>0?

1) x-y>0
2) xy>0

=>

The question x/y>0 is equivalent to xy > 0 after multiplying both sides of the inequality by y^2. It is equivalent to the condition 2).

Ans: B
Intern
Intern
Joined: 29 Jun 2014
Posts: 23
Own Kudos [?]: 3 [0]
Given Kudos: 51
Send PM
Re: The Ultimate Q51 Guide [Expert Level] [#permalink]
MathRevolution wrote:
1) As always, more developed questions combined with CMT 3 and 4 are being released. Look at the question below. This question, a 5051-level question including CMT 4(A), was released recently. To strengthen your skills for this type of questions, you need to know the relationship between variable approach and CMT.

If a and b are integers, is ab an odd?

1) a=0
2) b=1-a

==> In the original condition, there are 2 variables (a, b), and therefore C is most likely to be the answer. By solving con 1) and con 2), from a=0 and b=1, you get ab0*1=0=even, hence no, it is sufficient. Therefore, C is the answer. However, this is an integer question, one of the key questions. Thus, if you apply CMT 4 (A, B), if 1) a=0, you get ab=0 and it is always even, hence no, it is sufficient. Also, for con 2), from a+b=1=odd, and (a, b)=(odd, even), (even, odd), you get ab=even, hence yes, it is sufficient. Therefore, the answer is D. This question is related to CMT 4(B). In other words, con 1) is easy and con 2) is difficult, so you apply CMT 4(B) (If you get A and B easily, consider B).
Answer: D


MathRevolution wrote:
It is well-known that the way to find out approximation value of a positive integer n’s square root is following;
1st approximation: select a positive integer "a" and n is divided by a.
2nd approximation: a positive integer n’s square root is the average (arithmetic mean) of a quotient and divisor.
What is the approximation positive integer n’s square root, in terms of a and n?
A. (a2+n)/2a B. (a2+n)/2 C. (a2-n)/2a D. (a2+n)/a E. (a2+2n)/a
Answer: A

This is a thesis=like question. It is a very challenging 50-51 level question. N=aQ means √n=(a+Q)/2. Hence, the correct answer is A.


MathRevolution wrote:
Geometry questions are continuously increasing. Let's have a look at the recent question below.
Attachment:
CUBE.jpg


A cube has 4 as a side’s length. If A and B are midpoints of each side and C is a vertex of the cube, what is the length of AB?
A. 2√3
B. 3√6
C. √6
D. 3√2
E. 2√6

Answer: E

In a case of this question, you can just use Pythagoras' theorem twice. This type of geometry questions are likely to have been increasing since 2014. If AC^2=2^2+4^2=20, AB=x, x^2=2^2+AC^2=24 is derived. Then, x= √24= 2√6.
The answer is E.



Sent from my SM-A510F using GMAT Club Forum mobile app
GMAT Club Bot
Re: The Ultimate Q51 Guide [Expert Level] [#permalink]
   1  ...  14   15   16   17   18   19   20  ...  64   

Powered by phpBB © phpBB Group | Emoji artwork provided by EmojiOne